If Skiff's book is published this year, Professor Nguyen vows she will urge the dean to promote Skiff. Thus, if Skiff...

Matt on January 7, 2015

Hello

Why A and not D? D meets both requirements and A meets only one.

Replies
Create a free account to read and take part in forum discussions.

Already have an account? log in

Naz on January 9, 2015

This is a strengthen with sufficient premise question. Remember that a sufficient premise is sufficient for a conclusion, if and only if the existence of the premise guarantees or brings about the existence of the conclusion. Therefore, we need to find the premise that 100% guarantees the conclusion. The way you want to attack these answer choices is two-pronged. Ask yourself, does it strengthen? If it doesn't, then cross it out and continue to the next answer choice. If it does strengthen, however, then ask yourself whether or not the premise guarantees the conclusion.

Conclusion: "he will be promoted."

Why? We are told that if Skiff's book is published this year, then Professor Nguyen vows she will urge the dean to promote Skiff. We know that if Skiff's book is as important and as well written as Skiff claims, he will be promoted, because Nguyen will keep her promise and because if Nguyen recommends it, the dean will surely promote Skiff.

SO let's diagram this.

"If Skiff's book is published this year, Professor Nguyen vows she will urge the dean to promote Skiff."

P1: SBP ==> NVUDP
not NVUDP ==> not SBP

"if Nguyen recommends it, the dean will surely promote Skiff."

P2: NVUDP ==> DPS
not DPS ==> not NVUDP

"If Skiff's book is as important and as well written as Skiff claims, he will be promoted."

P3: I & WW ==> DPS
not DPS ==> not I or not WW

C: DPS

Answer choice (A): "Skiff's book will be published this year if it is as important as he claims it is."

(A) I ==> SBP
not SBP ==> not I

Does this strengthen the argument? Yes.

We want to strengthen the conclusion that Skiff will be promoted. Though we do not have both "I" and "WW," answer choice (A) still leads us to the conclusion that he will be promoted. We can connect answer choice (A) to "P1" like so: I ==> SBP ==> NVUDP to conclude: I ==> NVUDP, which we can then connect to "P2" like so: I ==> NVUDP ==> DPS to conclude: I ==> DPS, which means if we have answer choice (A), we can conclude that the dean will promote Skiff.

I understand your concern of answer choice (A) only discussing that Skiff's book is "as important as he claims it is," and not specifically discussing that it is "as well written as Skiff claims." However, this is irrelevant to our goal. We want to strengthen the conclusion, which ultimately is that Skiff will be promoted. Answer choice (A) helps us get there.

Now, continuing on with our Strengthen with Sufficient Premise technique: does answer choice (A) guarantee the conclusion? Yes.

As you can see in the above chain, we can - by using the transitive property - properly conclude that Skiff will be promoted.

Answer choice (D) is: "Skiff's book will not be published unless it as important and as well written as he claims it is."

We would rewrite this: If Skiff's book is published, then it is as important and as well written as he claim it is.

(D): SBP ==> I & WW
not I or not WW ==> not SBP

Does this strengthen the argument? No.

This is irrelevant to the argument. We can use (D) to connect to P3 like so: SBP ==> I & WW ==> DPS. However, we get stuck here. "DPS" is a necessary condition, and therefore, does not lead us to anything else. We want to prove that Skiff will be promoted. So, (D) is not our answer.

Hope that clears things up! Please let us know if you have any other questions.

Aidyn-Carlson on June 12, 2019

Hi can you re-explain why A and not D? D seems like the more logical answer because it includes both "important" "and well written" and the stimulus says skiff will not be published unless his material is important and well written.